\documentclass[a4paper,10pt]{article} \usepackage{myXsim} \usepackage{pgfplots} % \pgfplotsset{compat = newest} % \usepgfplotslibrary{external} % \tikzexternalize % Title Page \title{DM 4 \hfill \Var{Nom}} \tribe{2nd6} \date{À rendre pour lundi 4 avril 2022} \pagestyle{empty} \xsimsetup{ solution/print = false } \begin{document} \maketitle \begin{exercise}[subtitle={Information chiffrée}, points=4] Les questions suivantes n'ont pas de liens entre elles. \begin{enumerate} %- set evo_annuelle = random.randint(10, 60) / 10 %- set nbr_annee = random.randint(3, 6) \item Dans un pays, les prix augmentent de $\Var{evo_annuelle}\%$ par an. Bob a dormi pendant \Var{nbr_annee} ans. Quel sera le taux d'évolution des prix qu'il percevra? %- set evo1 = random.randint(5, 20) %- set evo2 = random.randint(5, 20) %- set evo3 = random.randint(30, 50) \item Une quantité a augmenté de $\Var{evo1}\%$ puis augmenté de $\Var{evo2}\%$ pour enfin diminuer de $\Var{evo3}\%$. Quel est le taux d'évolution global de cette quantité? %- set evo_direct = random.randint(30, 70) \item Les résultats du bac ont diminué de \Var{evo_direct}\%. Quel doit être le taux d'évolution des résultats pour qu'ils reviennent à leur niveau initial? %- set vf = random.randint(150, 300) %- set evo2_direct = random.randint(5, 30) \item Après une augmentation de \Var{evo2_direct}\%, le prix d'un velo est de \Var{vf}\euro. Quel était le prix de ce vélo avant cette augmentation? \end{enumerate} \end{exercise} \begin{solution} \begin{enumerate} %- set CM_global = round((1 + evo_annuelle/100)**nbr_annee, 3) \item Coefficient multiplicateur global: $(1 + \dfrac{\Var{evo_annuelle}}{100})^{\Var{nbr_annee}} \approx \Var{CM_global}$ %- set tx_global = round(CM_global - 1, 4) Taux d'évolution sur la période: $t = CM - 1 = \Var{CM_global} - 1 = \Var{tx_global} = \Var{tx_global*100}\%$. %- set CM_global = round((1 + evo1/100)*(1+evo2/100)*(1 - evo3/100), 3) \item Coefficient multiplicateur global: $(1 + \dfrac{\Var{evo1}}{100})\times(1 + \dfrac{\Var{evo2}}{100})\times (1 - \dfrac{\Var{evo3}}{100}) \approx \Var{CM_global}$ %- set tx_global = round(CM_global - 1, 4) Taux d'évolution sur la période: $t = CM - 1 = \Var{CM_global} - 1 = \Var{tx_global} = \Var{tx_global*100}\%$. %- set CM_direct = round(1 - evo_direct/100, 4) %- set CM_recip = round(1/CM_direct, 4) \item Coefficient multiplicateur $1 - \dfrac{\Var{evo_direct}}{100} = \Var{CM_direct}$ Coefficient multiplicateur réciproque: $\dfrac{1}{CM} = \dfrac{1}{\Var{CM_direct}} \approx \Var{CM_recip}$ %- set tx_recip = round(CM_recip - 1, 4) Taux d'évolution réciproque: $t = CM - 1 = \Var{CM_recip} - 1 = \Var{tx_recip} = \Var{tx_recip * 100} \%$ %- set CM_direct = round(1 + evo2_direct/100, 4) %- set vi = round(vf / CM_direct, 3) \item Une augmentation de $\Var{evo2_direct}\%$ signifie que la quantité au été multiplié par $\Var{CM_direct}$. Donc pour retrouver le prix initial, il faut diviser le prix final par $\Var{CM_direct}$ soit $\Var{vf} \times \dfrac{1}{\Var{CM_direct}} = \Var{vi}$. \end{enumerate} \end{solution} \begin{exercise}[subtitle={Statistiques}, points=2] \end{exercise} \begin{solution} \end{solution} \begin{exercise}[subtitle={Inéquations}, points=5] \begin{enumerate} \item Compléter le tableau suivant %- set m1, M1 = random.randint(-10, -5), random.randint(-5, 10) %- set M2 = random.randint(-10, 10) %- set M3 = random.randint(-10, 10) \begin{center} \renewcommand{\arraystretch}{3} \begin{tabular}{|p{5cm}|c|p{6cm}|p{3cm}|} \hline Phrase en français & Inégalité & Représentation sur la droite & Intervalle \\ \hline & $\Var{m1} < x \leq \Var{M1}$ & & \\ \hline & $x < \Var{M1}$ & & \\ \hline & & & $x \in \intOF{-\infty}{\Var{M3}}$\\ \hline \end{tabular} \end{center} \item Résoudre les inéquations suivantes et mettre les résultats sours forme d'un interval. \begin{multicols}{2} %- set a = random.randint(2, 10) %- set b = random.randint(2, 10) $\Var{a}x + \Var{b} < 0$ %- set a1 = random.randint(-10, -2) %- set b1 = random.randint(2, 10) %- set c1 = random.randint(2, 10) $\Var{a1}x - \Var{b1} \leq \Var{c1}$ \end{multicols} \end{enumerate} \end{exercise} \begin{solution} \begin{enumerate} \item pas de correction automatique \item \begin{align*} \Var{a}x + \Var{b} &< 0 \\ \Var{a}x &< -\Var{b} \\ \frac{\Var{a}}{\Var{a}}x &< \frac{-\Var{b}}{\Var{a}} \\ x &< \frac{-\Var{b}}{\Var{a}} \end{align*} Donc $x = \intOO{-\infty}{\frac{-\Var{b}}{\Var{a}}}$ \begin{align*} \Var{a1}x + \Var{b1} &\leq \Var{c1} \\ %- set d1 = c1 - b1 \Var{a1}x &\leq \Var{c1}-\Var{b1} \leq \Var{d1}\\ \frac{\Var{a1}}{\Var{a1}}x &\geq \frac{\Var{d1}}{\Var{a1}} \\ x &\geq \frac{\Var{d1}}{\Var{a1}} \end{align*} Donc $x = \intFO{\frac{\Var{d1}}{\Var{a1}}}{+\infty}$ \end{enumerate} \end{solution} \begin{exercise}[subtitle={Géométrie repérée}, points=2] \end{exercise} \begin{solution} \end{solution} \end{document}